Becky made a box in the shape of the triangular prism shown. If she were to wrap this box, how many square inches of wrapping paper would she need? Select one: O 225 square inches O 240 square inches O 156 square inches O 216 square inches

Becky Made A Box In The Shape Of The Triangular Prism Shown. If She Were To Wrap This Box, How Many Square

Answers

Answer 1

Answer:

216 square inches​

Explanation:

To find out how many square inches of wrapping paper she would need, calculate the total surface area of the triangular prism.

The triangular prism has five faces:

• Two equal triangles with base 6 in. and height 4 in.

,

• Two rectangles (Rectangle 1) of dimension 5 in. by 12 in.

,

• A rectangle of dimension 6 in. by 12 in.

Therefore:

[tex]\begin{gathered} \text{Surface Area}=2(\text{Area of Triangle)+2(Area of Rectangle 1)+Area of Rect. 2} \\ =2(\frac{1}{2}\times6\times4)+2(5\times12)+(6\times12) \\ =24+120+72 \\ =216\; in^2 \end{gathered}[/tex]

She would need 216 square inches​ of wrapping paper.


Related Questions

A full moon occurs every 30 days. If the last full moon occurred on a Friday, how many days will pass before a full moon occurs again on a Friday?

(I wrote the problem correctly, and its not 30)

Answers

Answer:

210 days

Step-by-step explanation:

7 x 30 = 210 days

Answer:

210

Step-by-step explanation:

A Friday occurs every 7 days.

A new moon occurs (approximately) every 30 days.

We need the smallest number that is a multiple of both 7 and 30.

We need the least common multiple (LCM) of 7 and 30.

This is the same as finding the least common denominator for two fractions that have denominators 7 and 30.

First, we find the prime factorizations of the two numbers.

7 = 7

30 = 2 × 3 × 5

The LCM is the product of common and not common factors with the larger exponent.

LCM = 2 × 3 × 5 × 7 = 210

Answer: 210

P.S. This is the correct answer for this problem, but the problem is stated incorrectly since a new moon occurs approximately every 29.5 days, so in reality, the true answer is not 210 days.

What is (9^6 x7^-9)^-4=

Answers

Answer:

[tex] \dfrac{7^{36}}{9^{24}} [/tex]

Step-by-step explanation:

(9^6 x7^-9)^-4=

[tex] = (9^6 \times 7^{-9})^{-4} [/tex]

[tex] = 9^{-24} \times 7^{36} [/tex]

[tex] = \dfrac{7^{36}}{9^{24}} [/tex]

What is the remainder when f(x) = x3 + 5x2 –32x – 7 is divided by x – 4? Show your work.

Answers

The remainder of f(x) is 385.

What are the 5 steps of long division?

The five steps of long division are given by:

Step 1: Divide

Step 2: Multiply

Step 3: Subtract

Step 4: Bring down

Step 5: Repeat

Given,

f(x) = x3 + 5x2 –32x – 7 is divided by x – 4

To find the Remainder

Now, The solution is:

             [tex]x^{2} -9x+98[/tex]

[tex]x-4\sqrt{x^3+5x^2-32x-7}\\[/tex]

         [tex]x^{3} - 4x^2[/tex]

        -     +

      -----------------------------

                [tex]9x^{2} -32x[/tex]

                [tex]9x^{2} +36x[/tex]

              -        -

         -------------------------------

                       98x -7

                       98x - 392

                       -       +

         ----------------------------------

             Remainder  385

Hence, The remainder of f(x) is 385.

Learn more about Long division Method at:

https://brainly.com/question/3935316

#SPJ1


On Wednesday, they pick a total of 89 kg of strawberries.
The mean mass of each strawberry is 22 g.
Work out the number of strawberries they picked.
Give your answer correct to the nearest 10

Answers

The number of strawberry they picked is about 4050.

What does math division mean?

Multiplication is the opposite of division. If 3 groups of 4 add up to 12, then 12 divided into 3 groups of equal size results in 4 in each group. Creating equal groups or determining how many people are in each group after a fair distribution is the basic objective of division.

89 kg = 89000 g conversion

89000=22=4045.45 ≈4045

{unit conversion}

≈4050

The number of strawberry {nearest lo} is about 4050

To learn more about division from given link

https://brainly.com/question/24662212

#SPJ9

Let g(x) = |2x - 10| find g(-3)

Answers

Answer:

g(-3) = 16

Step-by-step explanation:

To find g(-3), we need to plus -3 into the given function. So, g(-3) = |2(-3) - 10|. Now we solve using PEMDAS. Two times -3 is -6, then we have g(-3) = |-6-10| which is essentially g(-3) = |-16|. Lastly, we take the absolute value of |-16| which is just 16, therefore g(-3) = 16.

chegg a spherical ball is measured to have a radius of 5 mm, with a possible measurement error of 0.1 mm. use the differential to estimate the possible change in volume (in mm3) resulting from the error in measuring the radius.

Answers

The possible change in volume that results from the error in measuring the radius is 32. 06 millimeters

How to determine the volume of the sphere

It is important to note that the formula for volume of a sphere is expressed by;

Volume = 4/3 πr³

Given that;

r is the radius of the sphereπ has the value 3.14

Based on the information given, we have the measure of the radius to be 5 mm, with a possible measurement error of 0.1 mm

The original value of radius  = 5mm

Radius after the measurement error = 5 + 0. 1 = 5. 01mm

Let's substitute the values into the formula

Volume = 4/ 3 (3.14)(5)³

We then expand the bracket

Volume = 4/ 3 (392.5)

Volume = 523. 3 cubic millimeters

For radius of the measurement error = 5.01mm

Volume =  4/ 3 (3.14)(5.1)³

Then, expand the bracket

Volume = 4/3 (416.52)

Volume = 555. 4 cubic millimeters

Change in volume = 555. 4 - 523. 3 = 32. 06 cubic millimeters

Hence, the change in volume is given as 32. 06 cubic millimeters

Learn more about volume here:

https://brainly.com/question/1972490

#SPJ1

Determine whether the two figures are similar.

Answers

The two triangles are similar.

How to find similar triangles?

Similar triangles are triangles that have the same shape, but their sizes may vary.

Two triangles are said to be similar if their corresponding angles are congruent and the corresponding sides are in proportion .

The side lengths of two similar triangles are proportional. That is, if ΔABC is similar to ΔDEF , then the following equation holds:

AB / DE  = AC / DF = BC / EF

Hence,

6 / 3 = 10 / 5 = 8 / 4 = 2 / 1

This common ratio is called the scale factor .

The symbol ∼ is used to indicate similarity.

Therefore, the triangles are similar.

learn more on similar triangle here: https://brainly.com/question/22411257

#SPJ1

The ordered pair (a,b) satisfies the inequality y

A. If you subtract 6 from b, it will be less than or equal to a.

B. If you add 6 to a, it will be greater than b.

C. a is greater than b.

D. If you add 6 to a, it will be less than b.

Answers

The false statement about the statement is (a) If you subtract 6 from b, it will be less than or equal to a.

How to determine the true statement?

From the question, the inequality is given as

y > x - 6

The ordered pair is given as

(a, b)

The above ordered pair implies that

(x, y) = (a, b)

So, we have

x = a and y = b

This means that

b > a - 6

Subtract 6 from both sides

b - 6 > a - 6 - 6

Evaluate

b - 6 > a - 12

Using the above as a guide, we cannot determine the direct relationship between variables a and b.

However, it is false that b is less than a

Hence, the false statement is (a)

Read more about inequalities at

https://brainly.com/question/25275758

#SPJ1

Complete question

The ordered pair (a,b) satisfies the inequality y > x - 6

Which statement is NOT true?

A. If you subtract 6 from b, it will be less than or equal to a.

B. If you add 6 to a, it will be greater than b.

C. a is greater than b.

D. If you add 6 to a, it will be less than b.

Write an equation for the intervals of a parabola with x-intercepts at (2,0) and and (-5,0) that passes through the point (1, -18).

Help is always greatly appreciated.

Answers

[tex]\quad \huge \quad \quad \boxed{ \tt \:Answer }[/tex]

[tex]\qquad\displaystyle \tt \rightarrow \: y= 3 {x}^{2} + 9x - 30[/tex]

____________________________________

[tex] \large \tt Solution \: : [/tex]

The values of x where a parabola cuts the x - axis (y = 0) are the roots of the quadratic equation.

I.e -5 and 2 for the given problem.

and the equation can be represented as :

[tex]\qquad\displaystyle \tt \rightarrow \: y = a(x - x1)(x- x2)[/tex]

where, x1 and x2 are the roots of the quadratic equation, a is a constant value (depicting strech in curve)

Now, plug in the values :

[tex]\qquad\displaystyle \tt \rightarrow \: y= a(x- 2)(x - ( - 5))[/tex]

[tex]\qquad\displaystyle \tt \rightarrow \: y = a(x- 2)(x+ 5)[/tex]

[tex]\qquad\displaystyle \tt \rightarrow \: y = a( {x}^{2} + 5x - 2x - 10)[/tex]

[tex]\qquad\displaystyle \tt \rightarrow \: y= a( {x}^{2} + 3x - 10)[/tex]

Now, we need to find the value of a, for that let's use the coordinates of a point lying on the curve (1 , -18)

[tex]\qquad\displaystyle \tt \rightarrow \: - 18 = a( {1}^{2} + 3(1) - 10)[/tex]

[tex]\qquad\displaystyle \tt \rightarrow \: - 18 = a(1 + 3 - 10)[/tex]

[tex]\qquad\displaystyle \tt \rightarrow \: - 18 = a( - 6)[/tex]

[tex]\qquad\displaystyle \tt \rightarrow \: a = ( - 18) \div ( - 6)[/tex]

[tex]\qquad\displaystyle \tt \rightarrow \: a = 3[/tex]

Now, we got all required values. let's plug the value of a in equation, and we will get the required equation of parabola.

[tex]\qquad\displaystyle \tt \rightarrow \: y= 3( {x}^{2} + 3x - 10)[/tex]

[tex]\qquad\displaystyle \tt \rightarrow \: y= 3 {x}^{2} + 9x - 30[/tex]

Answered by : ❝ AǫᴜᴀWɪᴢ ❞

Answer:

[tex]\textsf{Factored form}: \quad f(x)=3(x-2)(x+5)[/tex]

[tex]\textsf{Standard form}: \quad f(x)=3x^2+9x-30[/tex]

Step-by-step explanation:

Factored form of a quadratic function

[tex]f(x)=a(x-p)(x-q)[/tex]

where:

p and q are the x-intercepts.a is some constant.

Given x-intercepts:

(2, 0)(-5, 0)

Substitute the given x-intercepts into the formula:

[tex]\implies f(x)=a(x-2)(x+5)[/tex]

To find a, substitute the given point (1, -18) into the equation and solve for a:

[tex]\implies -18=a(1-2)(1+5)[/tex]

[tex]\implies -18=a(-1)(6)[/tex]

[tex]\implies -6a=-18[/tex]

[tex]\implies a=3[/tex]

Therefore, the equation of the function in factored form is:

[tex]\boxed{ f(x)=3(x-2)(x+5)}[/tex]

Expand the brackets:

[tex]\implies f(x)=3(x^2+3x-10)[/tex]

[tex]\implies f(x)=3x^2+9x-30[/tex]

Therefore, the equation of the function in standard form is:

[tex]\boxed{f(x)=3x^2+9x-30}[/tex]

I have missing work may someone help me with my math

Answers

2)

[tex]w^2-3,\quad w=4[/tex]

When we say that w = 4, it means that we can rewrite the expression and instead of "w", we will put "4", so

[tex]w^2-3\Rightarrow4^2-3[/tex]

Now we have a numerical expression to solve, we can easily solve it!

[tex]\begin{gathered} 4^2-3 \\ \\ 4\cdot4-3 \\ \\ 16-3 \\ \\ 13 \end{gathered}[/tex]

Therefore, the final answer is

[tex]4^2-3=13[/tex]

5)

Here we will do the same thing, the "hard" part is solving the numeric expression, it will be a little bit harder than the 2).

[tex]3(6m-17),\quad m=5[/tex]

Again, repeat the same process, rewrite the expression, and instead of "m" you put "5"

[tex]3(6m-17)\Rightarrow3(6\cdot5-17)[/tex]

Again, another expression to simplify, remember that we always solve what is inside ( ) first, and we have a multiplication inside ( ) so we must solve it first

[tex]3(6\cdot5-17)=3(30-17)[/tex]

Now we solve the multiplication inside ( ) we can do the subctration

[tex]3(30-17)=3\cdot(13)[/tex]

The last step is just to solve another multiplication

[tex]3\cdot(13)=39[/tex]

Now we simplified everything we can have the final answer:

[tex]3(6\cdot5-17)=39[/tex]

6)

Here we have a division, but it's similar with 5) and 2), we have

[tex]\frac{2a}{3}+13,\quad a=15[/tex]

No secrets, repeat the process, but here, "a" will turn into "15", then

[tex]\frac{2a}{3}+13\Rightarrow\frac{2\cdot15}{3}+13[/tex]

We can do the multiplication at the numerator of the fraction

[tex]\frac{2\cdot15}{3}+13=\frac{30}{3}+13[/tex]

Now we can simplify the fraction, 30 divided by 3 is 10, then

[tex]\frac{30}{3}+13=10+13[/tex]

Now just do the sum and it's done!

[tex]10+13=23[/tex]

Hence the final answer is

[tex]\frac{2\cdot15}{3}+13=23[/tex]

ANSWERS:

1) 43

2) 13

3) 67

4) 12

5) 39

6) 23

Tea that sells for $2.00 a pound is mixed with a different tea that sells for
$5.00 a pound to produce 40 pounds of a custom blended tea that will
sell for $4.25 a pound. How many pounds of the less expensive tea
should be used in the mixture?
(1) 10
(2) 15
(3) 25
(4) 30

Pls show work pls show work pls show work pls show work

Answers

10 pounds of less expensive tea should be used in the mixture.

How to find the less expensive tea price?

Inferior tea price = $2.00

Superior tea price = $5.00

Blended tea price = $4.25

Solution:

Price of 40 pounds of a custom blended tea = $4.25 * 40 = $170

Let x be the inferior tea quantity and

(40 - x) be the superior tea quantity.

2x + 5(40 - x) = 4.25 * 40

2x + 200 -5x = 170

-3x = -30

x = 10

Therefore, 10 pounds of less expensive tea should be used in the mixture.

To learn more about mixture allegations, refer

https://brainly.in/question/1139725

#SPJ13

Rodney is flying on an airplane to puerto rico. his suitcase and the contents inside must weigh less than 50lbs. his suitcase weighs 4 pounds and the contents he wants to pack weigh 49 pounds. if each of his shirts weighs 0.75 pounds, write an inequality to represent the number of shirts he needs to remove to meet the flight requirements.

Answers

The inequality that represents, the number of shirts he needs to remove to meet the flight requirements is 0.75p + 50ibs  ≤ 49

What is a solution set to an inequality or an equation?

If the equation or inequality contains variable terms, then there might be some values of those variables for which that equation or inequality might be true. Such values are called solution to that equation or inequality.

Let the weight of Rodney suitcase be p

First the contents inside must weigh less than 50lbs.

x < 50

The suitcase weighs 4 pounds and the contents he wants to pack weigh 49 pounds.

0.75p < 49

Therefore, 0.75p + 50ibs  ≤ 49

The inequality that represents, the number of shirts he needs to remove to meet the flight requirements is 0.75p + 50ibs  ≤ 49

Learn more about inequalities here:

https://brainly.com/question/27425770

#SPJ1

The point B divides AC in the ratio 2:3.
Work out vector OC in terms of a and b.
Simplify your answer.

Answers

It is said that the ratio of point B to point AC is 2: 3.

This implies;

BC/AB = 3/2

BC = 3(b - a)/2

Thus;

AB + BC Equals AC

AC equals b-a plus 3(b-a)/2.

b-a - 3b/2 - 3a/2 = AC

AC = 5b/2 - 5a/2

AC = ⁵/₂(b - a) (b - a)

Using the same vector addition law, we can state that;

OC = AC – AO

OC = ⁵/₂(b - a) - (-a) (-a)

Given that giving OA is the inverse of -a, I used -a for AO.

Thus;

OC = 5b/2 - 5a/2 + a

OC = (⁵/₂)b - (³/₂)a

OC = ¹/₂(5b - 3a) (5b - 3a)

What is vector addition law?

Objects with both direction and magnitude are referred to be vectors. When two vectors have the same magnitude and direction, they are deemed to be one and the same. A line and an arrow are used to symbolize these geometrical objects. This arrow points in the direction of the vector, and the line's length indicates the vector's magnitude. These arrows, therefore, have a starting point and an ending point. Physical quantities like velocity, displacement, and acceleration are represented by vectors.

Notation A vector is frequently written in bold letters, such as an or b.

As seen on the right side, a vector may alternatively be expressed as the letters for its head and tail with an arrow above it.

Learn more about  vector addition law  visit:

https://brainly.com/question/14454864

#SPJ1

a playground is rectangular and the length is 7/8 miles. if the area is 8/20 what is the width?

Answers

Answer:

so the width is 16/35 miles

Step-by-step explanation:

Length - 7/8 miles

Area - 8/20 miles

we know from the equation that area = Length x width

we need to rearrange so the width = Area ÷Length

width = 8/20 ÷ 7/8

= 8/20 × 8/7

= 16/35

Solve for x Correct Answers PLS show work

Answers

Answer:

x = 8

Step-by-step explanation:

The angles shown are vertical angles

Vertical angles are congruent (equal to each other)

This means that 8x + 36 = 5x + 60

Using this equation we can solve for x

8x + 36 = 5x + 60  

==> subtract 36 from both sides

8x = 5x + 24

==> subtract 5x from both sides

3x = 24

==> divide both sides by 3

x = 8

Answer:

8x + 36 = 5x + 60

8x = 5x + 24

3x = 24

x = 8

-26, 174, 374, 574, …


Find the explicit and recursive formula.

Answers

The explicit formula and the recursive formula are aₙ₋₁ +  200 and the rest is mentioned below.

What does explicit and recursive formula mean?

A formula can be either recursive or explicit. The main difference between Recursive and Explicit is that Recursive formula gives the value of a specific term based on the previous term while Explicit formula gives the value of a specific term based on the position.

Given : a₁ =  -26, a₂ = 174, a₃ = 374, a₄= 574

The common difference that is d can be found out by

thus d = a₂ - a₁ = 174 -(- 26 ) = 174 + 26 = 200

And this is same for all the numbers of this particular sequence.

Thus aₙ = aₙ₋₁ + d

            = aₙ₋₁ +  200

Recursive formula :  aₙ₋₁ + d

That is the following sequence will have a common difference of 200 and the next 5th, 6th, 7th and nth term is going to be .

774 , 974 ... etc

To know more about arithmetic sequences visit:

https://brainly.com/question/8972906

#SPJ13

-15 = -3/4w solve for w and simplify your answer as much as possible

Answers

Answer:

w = 20

Step-by-step explanation:

a) Flip the equation.

-3/4 w = -15

b) Multiply both sides by 4/(-3).

(4/-3) * (-3/4 w) = (4/-3) * (-15)

w = 20

When driving up a certain hill, you rise 15 feet for every 1000 feet you drive forward. What is the slope of the rode?

Answers

slope of the rode is 0.015.

What is slope of line ?

Slope of line is the angle made by the line from positive x-axis in anticlockwise direction, it also denoted the steepness of the line.

The point with coordinate having same slope as with given coordinates can be plotted on the same line.

Here, it is given that :

When driving up a certain hill, we rise 15 feet for every 1000 feet that is :

for 15 feet up = 1000 feet forward

Now,

slope = upward distance / horizontal distance covered

slope = 15/1000

slope = 0.015

slope is a unitless quantity.

Therefore, slope of the rode is 0.015.

check and know more about slope here :

https://brainly.com/question/14914699

#SPJ1

11-(15g-13)=-6
what does G stand for

Answers

Answer:

g = 2

Step-by-step explanation:

11- (15g - 13) = - 6 ( subtract 11 from both sides )

- (15g - 13) = - 17 ( multiply both sides by - 1 )

15g - 13 = 17 ( add 13 to both sides )

15g = 30 ( divide both sides by 15 )

g = 2

Quadrilateral EFGH is inscribed inside a circle as shown below. Write a proof showing that angles H and F are supplementary.

Answers

Given

Find

showing that angles H and F are supplementary.

Explanation

Let angle F , which is subtended by minor arc GHE.

so , angle F = half of angle GHE by subtended angle theorem.

similarly , angle H is the angle subtended by major arc GFE.

so , angle H = half of the angle subtended by major arc.

since , total angle around J is 360 degree.

as we know , the sum of angle subtended by minor arc and major arc is 360 degree.

so , angle F + angle H = 1/2(360) = 180 degree.

Final Answer'

Hence , H and F are supplementary angles.

44+400 divided by (4+62)-24

Answers

Answer:

185

Step-by-step explanation:

Answer:

18

Step-by-step explanation:

250

244

18

10

are one of the answers

give this 5 stars if correct

You have exactly 10 coins with a total
value is $1. Three of the coins are
quarters. What are the remaining coins?
The

Answers

Answer:

2 dimes and 5 pennies

Step-by-step explanation:

2 dimes (d) is 10 and 5 pennies (p) is 5

75+p+d=100

75+p=80

80+d=100

Answer: 3 quarters, 2 dimes, 1 nickel.

Step-by-step explanation: 3 quarters are worth 75 cents, 2 dimes are worth 20 cents, and 1 nickel is worth 5 cents! Hope this helps

I NEED HELP WITH MY HOMEWORK

Answers

Chester made 9 sales and Vickie made 16 sales this week

Base salary of Vickie = $70

Vickie's commission per sale = $24

Base salary of Chester = $142

Chester's commission per sale = $9

Both people earned the same amount this week. Let x represent the number of sales made by Vickie, then sales made by Chester is x+7

Formulating the equation we get:

Base salary of Vickie + Vickie's commission per sale*Number of sales = Base salary of Chester + Chester's commission per sale*Number of sales

= 70 + 24x = 142 + 9(x+7)

70+24x = 142+9x+63

70+24x = 205+9x

15x = 135

x = 9

Chester sales = 9

Vickies sales = 9+7 = 16

Learn more about linear equations:

https://brainly.com/question/13738061

#SPJ1

A ball is thrown in the air the function H = 30t-5t^2 can be used to find the height (h) of the ball in meters after t seconds. how long does it take the ball to reach a height of 45 meters?

Answers

The most appropriate choice for Distance will be given by:

The ball takes [tex]3s[/tex] to reach a height of [tex]45[/tex] [tex]m[/tex].

What is Distance?

The length of the path an object takes without taking into account the direction of motion of the object is known as distance.

If [tex]s[/tex] is the speed of the object and [tex]t[/tex] is the time, then

Distance = [tex]s \times t[/tex]

Here,

[tex]H(t) = 30t - 5t^2[/tex], [tex]t[/tex] is the time in seconds

For finding the time taken by the ball to reach a height of [tex]45[/tex][tex]m[/tex], we need to substitute [tex]H(t) = 45[/tex]

Now putting [tex]H(t) = 45[/tex]

[tex]45 = 30t - 5t^2\\5t^2 - 30t+45 = 0\\ 5(t^2 -6t + 9)=0\\t^2 -6t+9=0\\t^2-3t-3t+9=0\\t(t-3)-3(t-3)=0\\(t-3)(t-3)=0\\t-3 = 0\\t = 3s[/tex]

So the ball takes [tex]3s[/tex] to reach a height of [tex]45[/tex] [tex]m[/tex].

To learn more about Distance, refer to the link:

https://brainly.com/question/26046491

#SPJ10

What percent of 933 is 391.83?

Answers

Answer:

42%

Tell me if you need expalining!

Step-by-step explanation:

The angle a lies between 0° and 90° and is such that
2 tan²a + sec²a = 5-4 tana

Show that

3 tan²a +4 tana -4 = 0
and hence find the exact value of tan a

Answers

Good.
Now we know this property
Sec^2 A = 1 + tan^2 A
Now lets replace the sec square term with this in the given question
We get,
3tan^2(A) + 1 = 5 - 4tanA
Hence 3tan^2A + 4tanA - 4 =0
Hence proved
Lets take tanA as a value x.
Thus 3x2+4x-4=0
=> 3x2 + 6x - 2x - 4 =0
=> 3x(x+2) -2(x+2) =0
=> (3x-2)(x+2)=0
x=2/3 or x=-2
But tanA cant have a negative value as 0Thus tanA=x= 2/3

Solve the inequality. Graph the solution. |8x−8|≤24

Answers

The solution of the given inequality would be x = 4. the graph of the given inequality is attached below.

What is a solution set to an inequality or an equation?

If the equation or inequality contains variable terms, then there might be some values of those variables for which that equation or inequality might be true. Such values are called solution to that equation or inequality. Set of such values is called solution set to the considered equation or inequality.

Given the inequality as |8x−8|≤24.

For solving the given expression,

|8x−8|≤24

We can consider the expression as;

8x−8 = 24

8x = 24 + 8

8x = 32

x = 32/8

x = 4

Hence, the solution of the given inequality would be x = 4.

Learn more about inequalities here:

https://brainly.com/question/27425770

#SPJ1

your company is producing special battery packs for the most popular toy during the holiday season. the life span of the battery pack is known to be normally distributed with a mean of 250 hours and a standard deviation of 20 hours. what is the probability that a randomly chosen battery pack lasts longer than 260 hours?

Answers

The probability that a randomly chosen battery pack lasts longer than 260 hours is 0.69.

What is a normal distribution?

The normal distribution is a probability distribution that is symmetric about the mean and demonstrates that data that are closer to the mean are more likely to occur than data that are farther from the mean.

Since the life span of the battery pack is known to be Normally distributed, we would apply the formula for normal distribution which is expressed as

z = (x - µ)/σ

Where

x = life spans of battery packs.

µ = mean life span

σ = standard deviation

From the information given,

µ = 250 hours

σ = 20 hours

The probability that a battery pack lasts longer than 260 hours. It is expressed as

P(x > 260) = 1 - P(x ≤ 260)

For x = 260

z = (260 - 250)/20 = 0.5

Looking at the normal distribution table, the probability corresponding to the z score is 0.69

To learn more about the normal distribution from the given link

https://brainly.com/question/4079902

#SPJ4

The probability that a randomly chosen battery pack lasts longer than 260 hours is 0.69.

What is a normal distribution?

The normal distribution is a probability distribution that is symmetric about the mean and demonstrates that data that are closer to the mean are more likely to occur than data that are farther from the mean.

Since the life span of the battery pack is known to be Normally distributed, we would apply the formula for normal distribution which is expressed as

z = (x - µ)/σ

Where

x = life spans of battery packs.

µ = mean life span

σ = standard deviation

From the information given,

µ = 250 hours

σ = 20 hours

The probability that a battery pack lasts longer than 260 hours. It is expressed as

P(x > 260) = 1 - P(x ≤ 260)

For x = 260

z = (260 - 250)/20 = 0.5

Looking at the normal distribution table, the probability corresponding to the z score is 0.69

To learn more about the normal distribution from the given link
brainly.com/question/4079902
#SPJ4

Which represents the inverse of the function f(x) = 4x?
Oh(x)=x+4
Oh(x)=x-4
Oh(x) = 3/4x
○ h(x) = 1/4x

Answers

Answer:

  (d)  h(x) = 1/4x

Step-by-step explanation:

You want to know the inverse of the function f(x) = 4x.

Inverse function

The inverse of y = f(x) will be the solution to x = f(y).

  x = f(y)

  x = 4y . . . . . . substitute the argument in the function definition

  x/4 = y . . . . . divide by 4 to solve for y

The inverse function is ...

  h(x) = (1/4)x

A plane (at point P) sees a bridge (at point B) at an angle of depression on 28 degrees. The horizontal distance between the plane and the bridge is 3000 feet. Explain which trigonometric equation can be used to solve for the height of the plane (segment PA). What is the height of the plane? You must show all work and calculations to receive full credit. Round your answer to the nearest foot.

Answers

The height of the plane PA is given as; 1591 ft

How to use trigonometric ratios?

There are different trigonometric ratios such as;

cos θ = adjacent/hypotenuse

sin θ = opposite/hypotenuse

tan θ = opposite/adjacent

Now, looking at the given image showing plane and bridge indicated by lines, we can use one of the trigonometric ratios to get the height of the plane PA as;

PA/3000 = tan B

now angle B = 28 degrees from alternate angles postulate. Thus;

PA/3000 = tan 28

PA = 3000 * tan 28

PA = 3000 * 0.5317

PA = 1591 ft

Read more about trigonometric ratios at; https://brainly.com/question/13276558

#SPJ1

[tex]tan 28^o = \frac{PA}{3000}[/tex]

tan 28° · 3000= PA

(use a scientific calculator)

1595.128= PA

height = 1595

Other Questions
Living things need __________ for thinking, eating and moving Mr. Charles drinks 8 glasses of water each day. Each glass contains 240ml. How manymillimeters of water does he drink in a week? State whether the Law of Detachment, the Law of Syllogism, or the Law of theContrapositive was used to draw the conclusion from the given statements. If it's aninvalid conclusion, write invalid.If you go fishing, then you will need to bring your fishing rod.If you go fishing, then you will need to bring bait.Therefore, if you bring your fishing rod, then you will need to bring bait.Law of DetachmentLaw of SyllogismLaw of ContrapositiveInvalid Conclusion Solve the following problem using substitution or elimination method. Two rental car companies are running specials this month. At Johnsons Rentals, customers will pay $50 to rent a mid-sized car for the first day, plus $2 for each additional day (x). At Martinez Rent-a-Car, the price for a mid-sized car is $30 for the first day and $4 for each additional day (x). How many additional days (x) would it cost the same to rent from either company? What is that cost?Let x represent the number of additional days you will rent a car and y represent the total cost of renting the car. Write a system to represent the situation, and solve to answer the question. use lagrange multipliers to prove that the rectangle with maximum area that has a given perimeter p is a square. let the sides of the rectangle be x and y and let f and g represent the area (a) and perimeter (p), respectively. find the following. Determine the x-intercepts of the graph represented by the following quadratic function. Recall that y = f(x).f(x) = x2 4x 21(x, y) = (smaller x-value)(x,y)= (larger x-value) In a newspaper, it was reported that yearly robberies in Springfield were down 40% to 126 in 2013 from 2012. How many robberies were there in Springfield in 2012? The length of a rectangular billboard is 9 meters more than the width. If the perimeter is 42 meters find the dimensions. Whats the Missing angle Need help with #9 please its due today hat is the value of ?The solution is What is the slope of the line on the graph policymakers who control monetary and fiscal policy and want to offset the effects on output of an economic contraction caused by a shift in aggregate supply could use policy to shift a. aggregate demand to the right. b. aggregate demand to the left. c. aggregate supply to the right. d. aggregate supply to the left. Multiply. State any excluded values. Simplify your answer. Type your answer in factored form. Albert invested money into the stock market, and the table represents his earnings. What type of function could be used to model his bank account as a function of time? Justify your answer.Week Balance ($)1 4282 6203 8124 1,004 This is an exponential function because there is a common difference in the balance between the weeks. This is an exponential function because there is a common ratio in the balance between the weeks. This is a linear function because there is a common difference in the balance between the weeks. This is a linear function because there is a common ratio in the balance between the weeks. which system of linear equations has the ordered pair (-4,-12)as it's solution? I tried to do this on my own and I got nine and they said it was incorrect so please help If a polynomial f(x) has a remainder of 8 when divided by x-4, what is f(4)?f(4) = name the four validities used to interrogate claims. if you were to question a studys four validities, provide a question you might ask related to each validity. a veterinary practice normally sells approximately 135 bottles of soloxine 0.3mg every 3 months (90 days). it takes 4 days to receive an order from the distributor from the time the inventory level is found to be low. what is the reorder point for soloxine?